Prove that $f(x)/x$ is increasing












0














Suppose $f(0)<0$ and that for all $x$, $f’’(x)>0$. Show that $f(x)/x$ is increasing on $(0,infty)$.



I tried to use the Taylor formula but it didn’t work. Thanks in advance for your help!










share|cite|improve this question




















  • 2




    The question in the title doesn't match with the one in the body of the question.
    – Kavi Rama Murthy
    Nov 20 '18 at 0:24










  • @KaviRamaMurthy thanks for reminding me
    – Jiu
    Nov 20 '18 at 0:24










  • @Bernard sorry I made a mistake in my questions
    – Jiu
    Nov 20 '18 at 0:25
















0














Suppose $f(0)<0$ and that for all $x$, $f’’(x)>0$. Show that $f(x)/x$ is increasing on $(0,infty)$.



I tried to use the Taylor formula but it didn’t work. Thanks in advance for your help!










share|cite|improve this question




















  • 2




    The question in the title doesn't match with the one in the body of the question.
    – Kavi Rama Murthy
    Nov 20 '18 at 0:24










  • @KaviRamaMurthy thanks for reminding me
    – Jiu
    Nov 20 '18 at 0:24










  • @Bernard sorry I made a mistake in my questions
    – Jiu
    Nov 20 '18 at 0:25














0












0








0


2





Suppose $f(0)<0$ and that for all $x$, $f’’(x)>0$. Show that $f(x)/x$ is increasing on $(0,infty)$.



I tried to use the Taylor formula but it didn’t work. Thanks in advance for your help!










share|cite|improve this question















Suppose $f(0)<0$ and that for all $x$, $f’’(x)>0$. Show that $f(x)/x$ is increasing on $(0,infty)$.



I tried to use the Taylor formula but it didn’t work. Thanks in advance for your help!







calculus analysis






share|cite|improve this question















share|cite|improve this question













share|cite|improve this question




share|cite|improve this question








edited Nov 20 '18 at 0:27









Bernard

118k639112




118k639112










asked Nov 20 '18 at 0:21









Jiu

435111




435111








  • 2




    The question in the title doesn't match with the one in the body of the question.
    – Kavi Rama Murthy
    Nov 20 '18 at 0:24










  • @KaviRamaMurthy thanks for reminding me
    – Jiu
    Nov 20 '18 at 0:24










  • @Bernard sorry I made a mistake in my questions
    – Jiu
    Nov 20 '18 at 0:25














  • 2




    The question in the title doesn't match with the one in the body of the question.
    – Kavi Rama Murthy
    Nov 20 '18 at 0:24










  • @KaviRamaMurthy thanks for reminding me
    – Jiu
    Nov 20 '18 at 0:24










  • @Bernard sorry I made a mistake in my questions
    – Jiu
    Nov 20 '18 at 0:25








2




2




The question in the title doesn't match with the one in the body of the question.
– Kavi Rama Murthy
Nov 20 '18 at 0:24




The question in the title doesn't match with the one in the body of the question.
– Kavi Rama Murthy
Nov 20 '18 at 0:24












@KaviRamaMurthy thanks for reminding me
– Jiu
Nov 20 '18 at 0:24




@KaviRamaMurthy thanks for reminding me
– Jiu
Nov 20 '18 at 0:24












@Bernard sorry I made a mistake in my questions
– Jiu
Nov 20 '18 at 0:25




@Bernard sorry I made a mistake in my questions
– Jiu
Nov 20 '18 at 0:25










1 Answer
1






active

oldest

votes


















1














$(frac {f(x)} x)'=frac {xf'(x)-f(x)} {x^{2}}$ so it is enough to show that $xf'(x)-f(x) > 0$. Now $(xf'(x)-f(x))'=xf''(x) > 0$ so $xf'(x)-f(x)$ is increasing and it is enough to observe that $0f'(0)-f(0)>0$.






share|cite|improve this answer





















    Your Answer





    StackExchange.ifUsing("editor", function () {
    return StackExchange.using("mathjaxEditing", function () {
    StackExchange.MarkdownEditor.creationCallbacks.add(function (editor, postfix) {
    StackExchange.mathjaxEditing.prepareWmdForMathJax(editor, postfix, [["$", "$"], ["\\(","\\)"]]);
    });
    });
    }, "mathjax-editing");

    StackExchange.ready(function() {
    var channelOptions = {
    tags: "".split(" "),
    id: "69"
    };
    initTagRenderer("".split(" "), "".split(" "), channelOptions);

    StackExchange.using("externalEditor", function() {
    // Have to fire editor after snippets, if snippets enabled
    if (StackExchange.settings.snippets.snippetsEnabled) {
    StackExchange.using("snippets", function() {
    createEditor();
    });
    }
    else {
    createEditor();
    }
    });

    function createEditor() {
    StackExchange.prepareEditor({
    heartbeatType: 'answer',
    autoActivateHeartbeat: false,
    convertImagesToLinks: true,
    noModals: true,
    showLowRepImageUploadWarning: true,
    reputationToPostImages: 10,
    bindNavPrevention: true,
    postfix: "",
    imageUploader: {
    brandingHtml: "Powered by u003ca class="icon-imgur-white" href="https://imgur.com/"u003eu003c/au003e",
    contentPolicyHtml: "User contributions licensed under u003ca href="https://creativecommons.org/licenses/by-sa/3.0/"u003ecc by-sa 3.0 with attribution requiredu003c/au003e u003ca href="https://stackoverflow.com/legal/content-policy"u003e(content policy)u003c/au003e",
    allowUrls: true
    },
    noCode: true, onDemand: true,
    discardSelector: ".discard-answer"
    ,immediatelyShowMarkdownHelp:true
    });


    }
    });














    draft saved

    draft discarded


















    StackExchange.ready(
    function () {
    StackExchange.openid.initPostLogin('.new-post-login', 'https%3a%2f%2fmath.stackexchange.com%2fquestions%2f3005740%2fprove-that-fx-x-is-increasing%23new-answer', 'question_page');
    }
    );

    Post as a guest















    Required, but never shown

























    1 Answer
    1






    active

    oldest

    votes








    1 Answer
    1






    active

    oldest

    votes









    active

    oldest

    votes






    active

    oldest

    votes









    1














    $(frac {f(x)} x)'=frac {xf'(x)-f(x)} {x^{2}}$ so it is enough to show that $xf'(x)-f(x) > 0$. Now $(xf'(x)-f(x))'=xf''(x) > 0$ so $xf'(x)-f(x)$ is increasing and it is enough to observe that $0f'(0)-f(0)>0$.






    share|cite|improve this answer


























      1














      $(frac {f(x)} x)'=frac {xf'(x)-f(x)} {x^{2}}$ so it is enough to show that $xf'(x)-f(x) > 0$. Now $(xf'(x)-f(x))'=xf''(x) > 0$ so $xf'(x)-f(x)$ is increasing and it is enough to observe that $0f'(0)-f(0)>0$.






      share|cite|improve this answer
























        1












        1








        1






        $(frac {f(x)} x)'=frac {xf'(x)-f(x)} {x^{2}}$ so it is enough to show that $xf'(x)-f(x) > 0$. Now $(xf'(x)-f(x))'=xf''(x) > 0$ so $xf'(x)-f(x)$ is increasing and it is enough to observe that $0f'(0)-f(0)>0$.






        share|cite|improve this answer












        $(frac {f(x)} x)'=frac {xf'(x)-f(x)} {x^{2}}$ so it is enough to show that $xf'(x)-f(x) > 0$. Now $(xf'(x)-f(x))'=xf''(x) > 0$ so $xf'(x)-f(x)$ is increasing and it is enough to observe that $0f'(0)-f(0)>0$.







        share|cite|improve this answer












        share|cite|improve this answer



        share|cite|improve this answer










        answered Nov 20 '18 at 0:27









        Kavi Rama Murthy

        50.1k31854




        50.1k31854






























            draft saved

            draft discarded




















































            Thanks for contributing an answer to Mathematics Stack Exchange!


            • Please be sure to answer the question. Provide details and share your research!

            But avoid



            • Asking for help, clarification, or responding to other answers.

            • Making statements based on opinion; back them up with references or personal experience.


            Use MathJax to format equations. MathJax reference.


            To learn more, see our tips on writing great answers.





            Some of your past answers have not been well-received, and you're in danger of being blocked from answering.


            Please pay close attention to the following guidance:


            • Please be sure to answer the question. Provide details and share your research!

            But avoid



            • Asking for help, clarification, or responding to other answers.

            • Making statements based on opinion; back them up with references or personal experience.


            To learn more, see our tips on writing great answers.




            draft saved


            draft discarded














            StackExchange.ready(
            function () {
            StackExchange.openid.initPostLogin('.new-post-login', 'https%3a%2f%2fmath.stackexchange.com%2fquestions%2f3005740%2fprove-that-fx-x-is-increasing%23new-answer', 'question_page');
            }
            );

            Post as a guest















            Required, but never shown





















































            Required, but never shown














            Required, but never shown












            Required, but never shown







            Required, but never shown

































            Required, but never shown














            Required, but never shown












            Required, but never shown







            Required, but never shown







            Popular posts from this blog

            Can a sorcerer learn a 5th-level spell early by creating spell slots using the Font of Magic feature?

            Does disintegrating a polymorphed enemy still kill it after the 2018 errata?

            A Topological Invariant for $pi_3(U(n))$